Chapter 22 - Disorders of Hemostasis, Chapter 23 - Disorders of Red Blood Cells, Chapter 26 - Disorders of Blood Flow, Chapter 26 - Disorders of Blood Pressure Regulation, Chapter 27 - Disorders of Cardiac Function, Chapter 27 - Heart failure and Cir...

¡Supera tus tareas y exámenes ahora con Quizwiz!

A 30-year-old woman who has given birth 12 hours prior is displaying signs and symptoms of disseminated intravascular coagulation (DIC). The client's husband is confused as to why a disease of coagulation can result in bleeding. Which of the nurse's following statements best characterizes DIC? A) "So much clotting takes place that there are no available clotting components left, and bleeding ensues." B) "Massive clotting causes irritation, friction, and bleeding in the small blood vessels." C) "Excessive activation of clotting causes an overload of vital organs, resulting in bleeding." D) "The same hormones and bacteria that cause clotting also cause bleeding."

A

A 31-year-old African American female who is in her 30th week of pregnancy has been diagnosed with peripartum cardiomyopathy. Which of the following statements best captures an aspect of peripartum cardiomyopathy? A) Her diagnosis might be attributable to a disordered immune response, nutritional factors, or infectious processes. B) Treatment is possible in postpartum women, but antepartum women are dependent on spontaneous resolution of the problem. C) Mortality exceeds 50%, and very few surviving women regain normal heart function. D) Symptomatology mimics that of stable angina and is diagnosed and treated similarly.

A

A 44-year-old female who is on her feet for the duration of her entire work week has developed varicose veins in her legs. What teaching point would her care provider be most justified in emphasizing to the woman? A) "Once you have varicose veins, there's little that can be done to reverse them." B) "Your varicose veins are likely a consequence of an existing cardiac problem." C) "If you're able to stay off your feet and wear tight stockings, normal vein tone can be reestablished." D) "The use of blood thinner medications will likely relieve the backflow that is causing your varicose veins."

A

Following a course of measles, a 5-year-old girl developed scattered bruising over numerous body surfaces and was diagnosed with immune thrombocytopenic purpura (ITP). As part of her diagnostic workup, blood work was performed. Which of the following results is most likely to be considered unexpected by the health care team? A) Increased thrombopoietin levels B) Decreased platelet count C) Normal vitamin K levels D) Normal leukocyte levels

A

A 6-year-old boy has been brought to the emergency department by ambulance after his mother discovered that his heart rate was "so fast I couldn't even count it." The child was determined to be in atrial flutter, and his mother is seeking an explanation from the health care team. Which of the following points should underlie an explanation to the mother? A) The child is experiencing a reentry rhythm in his right atrium. B) The resolution of the problem is dependent on spontaneous recovery and is resistant to pacing interventions. C) The child is likely to have a normal ECG apart from the rapid heart rate. D) The boy's atria are experiencing abnormal sympathetic stimulation.

A

A 68-year-old male complains to his family physician that when he tests his blood pressure using a machine at his pharmacy, his heart rate is nearly always very low. At other times, he feels that his heart is racing, and it also seems to pause at times. The man has also had occasionally light-headedness and a recent syncopal episode. What is this client's most likely diagnosis and the phenomenon underlying it? A) Sick sinus syndrome as a result of a disease of his sinus node and atrial or junctional arrhythmias B) Ventricular arrhythmia as a result of alternating vagal and sympathetic stimulation C) Torsade de pointes as a result of disease of the bundle of His D) Premature atrial contractions that vacillate between tachycardic and bradycardic episodes as a consequence of an infectious process

A

A 71-year-old male patient with a history of myocardial infarction (MI) and peripheral vascular disease (PVD) has been advised by his family physician to begin taking 81 mg aspirin once daily. Which of the following statements best captures an aspect of the underlying rationale for the physician's suggestion? A) Platelet aggregation can be precluded through inhibition of prostaglandin production by aspirin. B) Aspirin helps to inhibit ADP action and minimizes platelet plug formation. C) Aspirin can reduce unwanted platelet adhesion by inhibiting TXA2 synthesis. D) Aspirin inhibits the conversion of fibrinogen into fibrin and consequent platelet plug formation

A

A client with a diagnosis of atrophic gastritis and consequent pernicious anemia is receiving high oral doses of vitamin B12. Which of the following changes would be most likely expected by his care provider at the completion of his treatment? A) Decreased mean corpuscular volume B) Increased serum bilirubin C) Increased folic acid levels D) Decreased free heme levels

A

A family physician is performing patient teaching about the influenza virus with each patient who has come to the clinic to receive that year's vaccine. Which of the following statements by the patient best reflects an accurate understanding of the flu virus? A) "I could come down with viral or bacterial pneumonia as a result of a bad flu bug." B) "I know my vaccination is especially important since there aren't any drugs that can treat the flu once I get sick with it." C) "The emphasis on bundling up, staying warm, and drinking lots of fluids is outdated and actually ineffective." D) "Like all vaccines, it is ideal if everyone in a population gets immunized against the flu."

A

A female older adult client has presented with a new onset of shortness of breath, and her physician has ordered measurement of her brain natriuretic peptide (BNP) levels along with other diagnostic tests. What is the most accurate rationale for the physician's choice of blood work? A) BNP is released as a compensatory mechanism during heart failure, and measuring it can help differentiate the client's dyspnea from a respiratory pathology. B) BNP is an indirect indicator of the effectiveness of the renin-angiotensin-aldosterone (RAA) system in compensating for heart failure. C) BNP levels correlate with the client's risk of developing cognitive deficits secondary to heart failure and consequent brain hypoxia. D) BNP becomes elevated in cases of cardiac asthma, Cheyne-Stokes respirations, and acute pulmonary edema, and measurement can gauge the severity of pulmonary effects.

A

A midwife who is providing care for a woman during her first pregnancy is assessing for intrauterine growth retardation (IUGR) during an early prenatal checkup. Which of the following questions best addresses the risks for IUGR? A) "What does your typical diet look like over the course of a day?" B) "What is the highest level of education that you've finished?" C) "Are there many people in your life that you can count on for help and support?" D) "How would you describe your mood since you've been pregnant?"

A

A nurse educator in a geriatric medicine unit of a hospital is teaching a group of new graduates specific assessment criteria related to heart failure. Which of the following assessment criteria should the nurses prioritize in their practice? A) Measurement of urine output and mental status assessment B) Pupil response and counting the patient's apical heart rate C) Palpation of pedal (foot) pulses and pain assessment D) Activity tolerance and integumentary inspection

A

A nurse is providing care for a patient who has been admitted with a newly diagnosed bilateral pleural effusion. Which of the following findings from the nurse's initial assessment of the patient is incongruent with the patient's diagnosis and would require further investigation? A) The client complains of sharp pain exacerbated by deep inspiration. B) The client's breath sounds are diminished on auscultation. C) Pulse oximetry indicates that the client is hypoxemic. D) The client complains of dyspnea and increased work of breathing.

A

A nurse who provides care in a geriatric subacute medicine unit of a hospital has noted that a large number of patients receive -adrenergic blocking medications such as metoprolol. Which of the following statements best conveys an aspect of the use of beta-blockers? A) They can be used to treat supraventricular arrhythmias and decrease automaticity by depressing phase 4 of the action potential. B) They inhibit the potassium current and repolarization, extending the action potential and refractoriness. C) They counteract arrhythmias and tachycardias by increasing vagal stimulation. D) They decrease myocardial oxygen demand by blocking the release of intracellular calcium ions.

A

A patient arrives at the ED complaining of numbness in the left lower leg. Upon assessment, the nurse finds the lower left leg to be cold to touch, pedal and posterior tibial pulses nonpalpable, and a sharp line of paralysis/paresthesia. The nurse's next action is based on the fact that A) acute arterial occlusion is a medical emergency requiring immediate intervention to restore blood flow. B) submersion in a whirlpool with warm water will improve the venous blood flow And restore pulses. C) the immediate infusion of tissue plasminogen activator (tPA) will not correct the problem and should only be used for CVAs. D) administration of an aspirin and sublingual nitroglycerin will vasodilate the artery to restore perfusion.

A

A patient is reading a brochure on atherosclerosis while in the waiting room of medical clinic. Which of the following excerpts from the educational brochure warrants correction? A) "Because smoking causes a permanent increase in your risk of heart disease, it's best not to start." B) "All things being equal, men have a higher risk of coronary heart disease than perimenopausal women." C) "High blood pressure often accompanies, or even causes, clogging of the arteries." D) "Every bit that you can lower your cholesterol means that you'll have a lower risk of developing heart disease."

A

A patient with small cell lung cancer (SCLC) has developed a paraneoplastic syndrome called Cushing syndrome. Based on this new complication, the nurse will likely assess which of the following clinical manifestations of Cushing syndrome? A) Weight gain, moon face, buffalo hump, and purple striae on the abdomen B) Bilateral edema in the arms, swollen face, and protruding eyes C) Severe bone/joint pain, nausea/vomiting, and polyuria D) Tetany, new-onset seizure activity, emotional lability, and extrapyramidal symptoms

A

A patient, who is experiencing some angina associated with atrial tachycardia, has been placed on verapamil (Calan), a calcium channel blocker. Knowing that this medication blocks the slow calcium channels, thereby depressing phase 4 and lengthening phases 1 and 2 action potential, the nurse should assess this patient for which of the following adverse reactions? A) Bradycardia B) Ventricular tachycardia C) Sudden cardiac death D) Increased cardiac output

A

A pediatrician is teaching a group of medical students about some of the particularities of heart failure in children as compared with older adults. Which of the physician's following statements best captures an aspect of these differences? A) "You'll find that in pediatric patients, pulmonary edema is more often interstitial rather than alveolar, so you often won't hear crackles." B) "Because of their higher relative blood volume, jugular venous distention is a better assessment technique for suspected heart failure in young patients." C) "Signs and symptoms in children may sometimes mimic those of shock, with a low blood pressure and high heart rate." D) "Fever is a sign of heart failure in children that you are unlikely to see in older adults."

A

A physician is explaining to a group of medical students the concept of Virchow triad as it applies to venous thrombosis. Which of the following clinical observations of a 50-year-old male client is most likely unrelated to a component of Virchow triad? A) The man has decreased cardiac output and an ejection fraction of 30%. B) The man's prothrombin time and international normalized ratio (INR) are both low. C) The man has a previous history of a dissecting aneurysm. D) There is bilateral, brown pigmentation of his lower legs.

A

A physician is providing care for several patients on a medical unit of a hospital. In which of the following patient situations would the physician most likelyrule out hypertension as a contributing factor? A) A 61-year-old man who has a heart valve infection and recurrent fever B) An 81-year-old woman who has had an ischemic stroke and has consequent one-sided weakness C) A 44-year-old man awaiting a kidney transplant who requires hemodialysis three times per week D) A 66-year-old woman with poorly controlled angina and consequent limited activity tolerance

A

A premature infant on mechanical ventilation has developed bronchopulmonary dysplasia (BPD) and is showing signs and symptoms of hypoxemia, low lung compliance, and respiratory distress. Which of the following is the most likely contributor to the infant's present health problem? A) High-inspired oxygen concentration and injury from positive-pressure ventilation B) Failure to administer corticosteroids to the infant in utero C) Insufficient surfactant production and insufficient surfactant therapy D) Insufficient supplemental oxygen therapy

A

All of the following interventions are ordered stat. for a patient stung by a bee who is experiencing severe respiratory distress and faintness. Which priority intervention will the nurse administer first? A) Epinephrine (Adrenalin) B) Normal saline infusion C) Dexamethasone (Decadron) D) Diphenhydramine (Benadryl)

A

An IV drug abuser walks into the ED telling the nurse that, "he is sick." He looks feverish with flushed, moist skin; dehydrated with dry lips/mucous membranes; and fatigued. The assessment reveals a loud murmur. An echocardiogram was ordered that shows a large vegetation growing on his mitral valve. The patient is admitted to the ICU. The nurse will be assessing this patient for which possible life-threatening complications? A) Systemic emboli, especially to the brain B) Petechial hemorrhages under the skin and nail beds C) GI upset from the massive amount of antibiotics required to kill the bacteria D) Pancreas enlargement due to increased need for insulin secretion

A

As part of the diagnostic workup for a male client with a complex history of cardiovascular disease, the care team has identified the need for a record of the electrical activity of his heart, insight into the metabolism of his myocardium, and physical measurements and imaging of his heart. Which of the following series of tests is most likely to provide the needed data for his diagnosis and care? A) Echocardiogram, PET scan, ECG B) Ambulatory ECG, cardiac MRI, echocardiogram C) Serum creatinine levels, chest auscultation, myocardial perfusion scintigraphy D) Cardiac catheterization, cardiac CT, exercise stress testing

A

During a routine physical exam for a patient diagnosed with hypertension, the nurse practitioner will be most concerned if which of the following assessments are found? A) Noted hemorrhages and microaneurysms during evaluation of the internal eye B) Unable to feel vibrations when a tuning fork is placed on the skull C) Inability to locate the kidneys with deep palpation to the abdomen D) Slight increase in the number of varicose veins noted bilaterally

A

Emergency medical technicians respond to a call to find an 80-year-old man who is showing signs and symptoms of severe shock. Which of the following phenomena is most likely taking place? A) The man's - and -adrenergic receptors have been activated, resulting in vasoconstriction and increased heart rate. B) Hemolysis and blood pooling are taking place in the man's peripheral circulation. C) Bronchoconstriction and hyperventilation are initiated as a compensatory mechanism. D) Intracellular potassium and extracellular sodium levels are rising as a result of sodium-potassium pump failure

A

Knowing the high incidence and prevalence of heart failure among the elderly, the manager of a long-term care home has organized a workshop on the identification of early signs and symptoms of heart failure. Which of the following teaching points is most accurate? A) "Displays of aggression, confusion, and restlessness when the resident has no history of such behavior can be a sign of heart failure." B) "Heart failure will often first show up with persistent coughing and lung crackles." C) "Residents in early heart failure will often be flushed and have warm skin and a fever." D) "Complaints of chest pain are actually more often related to heart failure than to myocardial infarction."

A

Misinterpreting her physician's instructions, a 69-year-old woman with a history of peripheral artery disease has been taking two 325 mg tablets of aspirin daily. How has this most likely affected her hemostatic status? A) Irreversible acetylation of platelet cyclooxygenase activity has occurred. B) The patient's prostaglandin (TXA2) levels are abnormally high. C) She is at risk of developing secondary immune thrombocytopenic purpura (ITP). D) The binding of an antibody to platelet factor IV produces immune complexes.

A

The first-time parents of an infant girl 2 days postpartum are distressed at the jaundiced appearance of her skin and are eager for both an explanation and treatment for the problem. Which of the following responses by their physician is most accurate? A) "Your daughter's young liver is unable to get rid of the waste products from old red blood cells." B) "Because your daughter's kidneys are so small, they have a hard time getting rid of the wastes that are always accumulating in her blood." C) "Nearly half of all infants have this problem, and while it is distressing to look at, it is largely harmless and will resolve in time." D) "This is a sign that your baby needs more milk than she is currently getting, and increased breast-feeding will act to flush these pigments out of her system."

A

The mother of a 7-year-old boy who has recently been diagnosed with childhood asthma has come to the education center to learn more about her son's condition. Which of the following teaching points is most justifiable? A) "Research has shown that viruses may actually be a factor in many children's asthma." B) "The most reliable indicator that your child is having an asthma attack is audible wheezing." C) "Steroids that your child can inhale will likely be the first line of defense." D) "Your son will likely need to limit or avoid exercise and sports."

A

The nurse practitioner working in an overnight sleep lab is assessing and diagnosing patients with sleep apnea. During this diagnostic procedure, the nurse notes that a patient's blood pressure is 162/97. The nurse explains this connection to the patient based on which of the following pathophysiological principles? A) During apneic periods, the patient experiences hypoxemia that stimulates chemoreceptors to induce vasoconstriction. B) When the patient starts to snore, his epiglottis is closed over the trachea. C) When the airway is obstructed, specialized cells located in the back of the throat send signals to the kidney to increase pulse rate. D) When airways are obstructed, the body will retain extracellular fluid so that this fluid can be shifted to intravascular space to increase volume.

A

When discussing the sequence of clot dissolution, the science instructor will talk about which item that begins the process? A) Plasminogen B) Dabigatran C) Platelets D) 2-plasmin inhibitor

A

Which of the following hypertensive individuals is most likely to have his or her high blood pressure diagnosed as secondary rather than essential? A) A 51-year-old male who has been diagnosed with glomerulonephritis B) An African American man who leads a sedentary lifestyle C) A 69-year-old woman with a diagnosis of cardiometabolic syndrome D) A 40-year-old smoker who eats excessive amounts of salt and saturated fats

A

Which of the following medications will likely be prescribed for a patient with elevated LDL and triglyceride levels? A) Zocor (simvastatin), an HMG-CoA reductase inhibitor or "statin" B) Cholestyramine (Questran), a bile acid sequestrant C) Nicotinic acid (Niacin), a B vitamin D) Fenofibrate (Tricor), a fibric acid

A

Which of the following patients who presented to a walk-in medical clinic is most likely to be diagnosed with a rhinosinusitis rather than a common cold? A) A man complaining of general fatigue, a headache, and facial pain with a temperature of 100.9°F B) A woman presenting with malaise, lethargy, and copious nasal secretions C) A man with a dry, stuffy nasopharynx, a sore throat, and temperature of 98.9°F D) A woman complaining of generalized aches and who has a hoarse voice and reddened, painful upper airways

A

Which of the following patients would be most likely to be experiencing an increase in renal erythropoietin production? A) A 71-year-old smoker admitted to hospital with exacerbation of his chronic obstructive pulmonary disease (COPD) B) A 70-year-old woman admitted with dehydration secondary to an overdose of her potassium-wasting diuretic C) A 68-year-old man with a long-standing diagnosis of polycythemia vera D) A 21-year-old man with acute blood loss secondary to a motor vehicle accident 3 hours prior

A

Which of the following statements describes phase 4 of the action potential of cells in the sinoatrial (SA) node? A) A slow depolarization occurs when Na+ is transported out of the cell and K+ moves back in, resulting in resting membrane potential. B) The cells are capable of responding to a greater than normal stimulus before the resting membrane potential is reached. C) The fast sodium channels in the cellular membranes close, causing an abrupt decrease in intracellular positivity. D) Potassium permeability is allowing the cell membrane to remain depolarized, and Ca2+ channel opens moving Ca2+ back into the cell.

A

The nurse caring for a male child with respiratory problems is concerned he may be developing respiratory failure. Upon assessment, the nurse knows that which of the following are clinical manifestations of respiratory failure? Select all that apply. A) Severe accessory muscle retractions B) Nasal flaring C) Grunting on expiration D) Inspiratory wheezes heard E) Swollen glottis

A B C

A 78-year-old male patient has undergone a total knee replacement. He just does not feel like getting out of bed and moving around. After 3 days of staying in bed, the physical therapist encourages him to get out of bed to the chair for meals. He starts to complain of dizziness and light-headedness. These symptoms are primarily caused by which of the following pathophysiological principles? Select all that apply. The patient('s) A) is experiencing a reduction in plasma volume. B) peripheral vasoconstriction mechanism has failed. C) is so stressed that he is releasing too many endorphins. D) is still bleeding from the surgical procedure. E) has lost all of his muscle tone.

A B

A COPD patient asks the nurse what medications are prescribed to help his breathing. The nurse, looking at the list of medications, will educate the patient about which of the following medications to help his COPD in the long term? Select all that apply. A) Salmeterol (Serevent), a bronchodilator B) Tiotropium (Spiriva), anticholinergic C) Alprazolam (Xanax), a benzodiazepine D) Sildenafil (Viagra), a vasodilator E) Ketorolac (Toradol), an NSAID

A B

A newly diagnosed leukemia patient begins hemorrhaging from every orifice. The physician is concerned that the patient has developed disseminated intravascular coagulation (DIC). The nurse should anticipate which of the following orders to be prescribed for this patient? Select all that apply. A) Transfuse 2 units of platelets. B) Transfuse fresh frozen plasma. C) Give aspirin twice per day. D) Administer IV Toradol stat. E) Place in reverse isolation.

A B

The neonatologist suspects an infant has developed sepsis with multiorgan system illness. The nurse caring for this infant will note which of the assessment findings support this diagnosis. Select all that apply. A) Decreasing BP with increase in heart rate indicative of shock B) Prolonged PT and PTT and decrease in platelet count C) Frequent voiding of a small amount of light-colored urine D) Bilateral warm feet but pedal pulses hard to palpate E) Positive Moro reflex when loud noise made at crib side

A B

A patient arrives in the ED after an automobile accident. Which of the following clinical manifestations lead the nurse to suspect a pneumothorax? Select all that apply. A) Respiratory rate 34 B) Asymmetrical chest movements, especially on inspiration C) Diminished breath sounds over the painful chest area D) Pulse oximetry 98% E) ABG pH level of 7.38

A B C

Following cardiac surgery, the nurse suspects the patient may be developing a cardiac tamponade. Which of the following clinical manifestations would support this diagnosis? Select all that apply. A) Muffled heart tones B) Narrowed pulse pressure C) Low BP—84/60 D) Heart rate 78 E) Bounding femoral pulse

A B C

Two years after chemotherapy and radiation therapy for lung cancer, a 72-year-old patient notices that he seems to be extremely tired all the time. The physician suspects the patient may have developed aplastic anemia. The nurse assessing the patient will likely find which of the following clinical manifestations of aplastic anemia? Select all that apply. A) Complaints of weakness and fatigue B) Small spots of skin hemorrhages over the entire body C) Excess bleeding from gums and nose D) Spoon-shaped deformity of the fingernails E) Hemolysis from renal dialysis treatments.

A B C

Following a ST-segment myocardial infarction (STEMI), the nurse should be assessing the patient for which of the following complications? Select all that apply. A) Large amount of pink, frothy sputum and new onset of murmur B) Tachypnea with respiratory distress C) Frequent ventricular arrhythmia unrelieved with amiodarone drip D) Complaints of facial numbness and tingling E) Enhanced renal perfusion as seen as an increase in urine output

A B C D

Following coronary bypass graft (CABG) surgery for a massive myocardial infarction (MI) located on his left ventricle, the ICU nurses are assessing for clinical manifestations of cardiogenic shock. Which of the following assessment findings would confirm that the client may be in the early stages of cardiogenic shock? Select all that apply. A) Decreasing mean arterial pressure (MAP) B) Low BP reading of 86/60 C) Urine output of 15 mL last hour D) Low pulmonary capillary wedge pressure (PCWP) E) Periods of confusion

A B C E

A 66-year-old client's echocardiogram report reveals a hypertrophied left ventricle. The health care provider suspects the client has aortic stenosis. Which of the following clinical manifestations would be observed if this client has aortic stenosis? Select all that apply. A) Decrease in exercise tolerance B) Exertional dyspnea C) Palpitations D) Syncope E) Heartburn

A B D

A client has many residual health problems related to compromised circulation following recovery from septic shock. The nurse knows that which of the following complications listed below are a result of being diagnosed with septic shock and therefore should be assessed frequently? Select all that apply. A) Profound dyspnea due to acute respiratory distress syndrome B) Atelectasis resulting in injury to endothelial lining of pulmonary vessels, which allows fluid/plasma to build up in alveolar spaces C) Formation of plaque within vessels supplying blood to the heart causing muscle damage and chest pain D) Acute renal failure due to decreased/impaired renal perfusion as a result of low BP E) Flushed skin and pounding headache that coincides with each heart beat

A B D

A nurse is providing care for a client who has been admitted to a medical unit with a diagnosis of bronchiectasis. Which of the following signs and symptoms should the nurse expect to find during physical assessment of the client and the review of the client's history? Select all that apply. A) Recurrent chest infections B) Production of purulent sputum C) A barrel chest D) Low hemoglobin levels E) Recent surgery

A B D

A patient with pulmonary hypertension may display which of the following clinical manifestations? Select all that apply. A) Shortness of breath B) Decreased exercise tolerance C) Nasal flaring D) Grunting on expiration E) Swelling (edema) of the legs and feet

A B E

Which of the following nursing interventions would be a priority when caring for a newborn who is receiving phototherapy for high bilirubin levels? Select all that apply. A) Frequent monitoring of temperature B) Keeping diapers dry and clean C) Maintaining oral intake to prevent dehydration D) Putting lotion on his skin frequently to prevent drying/cracking of skin E) Applying sunscreen to prevent ultraviolet radiation

A C

Which of the following patients would be at risk for developing nonthrombocytopenic purpura? Select all that apply. A) A child adopted from India and displaying malaise, lethargy, and petechiae all over the body B) A 73-year-old patient admitted with concussion that resulted from a fall C) A 55-year-old patient diagnosed with Cushing disease displaying bruises, weight gain with a buffalo hump, and moon face D) A 15-year-old insulin-dependent diabetic with hypoglycemia displaying irritability with headaches and tachycardia E) A pregnant mother experiencing headaches and proteinuria

A C

A nurse working on a gerontology unit notes that the majority of the clients on the unit are prescribed antihypertensive medications. When it comes to the aging process, which of the following phenomena are primarily the contributing factors to hypertension in the elderly population? Select all that apply. A) Stiffening of large arteries like the aorta B) Increased sensitivity of the renin-angiotensin-aldosterone system C) Decreased baroreceptor sensitivity and renal blood flow D) Increased peripheral vascular resistance E) Increase in renal perfusion

A C D

A patient who has been on a high-protein diet comes to the emergency department with respiratory symptoms. Upon analysis of arterial blood gases (ABGs), the patient is diagnosed with hypercapnia. The nurse will note the ABG results that confirm this diagnosis include: Select all that apply. A) pH 7.31 (normal 7.35 to 7.45). B) PO2 of 97%. C) PCO2 of 58 mm Hg (normal 38 to 42). D) Serum HCO3of -33 mEq/L (normal 22 to 28). E) Serum K+ (potassium)of 3.6 mmol/L (normal 3.5 to 5.0).

A C D

A premature infant who is receiving care in a neonatal intensive care unit (NICU) has just been identified as having necrotizing enterocolitis (NEC). Of the following clinical manifestations, identify those most likely to contribute to the diagnosis of NEC. Select all that apply. A) Feeding intolerance B) Inability to pass stool within the first 10 days of life C) Hard, taut abdomen with increasing distention D) Blood noted in stools E) Hypoactive bowel sounds on right lower quadrant

A C D

During a prenatal education class, a participant has related a story about how her friend's infant died of sudden infant death syndrome (SIDS). What can the educator tell the group about how they can prevent SIDS when they have their babies? Select all that apply. A) "The best sleeping position for your baby is on his back." B) "Children are at particular risk of SIDS when they have a cold or flu, so these times require extra vigilance." C) "Using drugs during pregnancy has been shown to be associated with SIDS after birth, which is one more reason for mothers to avoid them." D) "It's important if anyone in your home smokes to make sure they only do it outside." E) "The exact cause of SIDS still isn't known, so there's little that you can do to prevent this tragic event."

A C D

Which of the following clients are displaying known risk factors for the development of pulmonary emboli? Select all that apply. A client who is: A) immobilized following orthopedic surgery. B) experiencing impaired Cl- and Na+ regulation. C) taking amiodarone for the treatment of premature ventricular contractions. D) a smoker and who takes oral contraceptives. E) undergoing radiation therapy for the treatment of breast cancer.

A D

While assessing a premature infant born at 25 weeks' gestation, the neonatal intensive care unit (NICU) nurse would suspect which diagnosis when the infant displays poor muscle tone, apnea, and a new onset of somnolence? A) Hydrocephalus B) Airway obstruction C) Intraventricular hemorrhage D) Sepsis

C

A 13-year-old African American boy comes to the ER complaining of fatigue and a rapid heartbeat. In conversation with the father, it becomes apparent to you that the boy has grown 2 inches in the previous 5 months. What is the first problem the health care team would attempt to rule out? A) Sickle cell anemia B) Iron deficiency anemia C) Thalassemia D) Aplastic anemia

B

A 13-year-old boy has had a sore throat for at least a week and has been vomiting for 2 days. His glands are swollen, and he moves stiffly because his joints hurt. His parents, who believe in "natural remedies," have been treating him with various herbal preparations without success and are now seeking antibiotic treatment. Throat cultures show infection with group A streptococci. This child is at high risk for A) myocarditis. B) mitral valve stenosis. C) infective endocarditis. D) vasculitis.

B

A 16-year-old adolescent who received a kidney transplant at the age of 10 has recently developed a trend of increasing BP readings. Of the following list of medications, which may be the primary cause for the development of hypertension? A) Furosemide (Lasix) B) Cyclosporine (Sandimmune) C) Isotretinoin (Accutane) D) Hydrochlorothiazide (Hydrodiuril)

B

A 3-year-old boy has developed croup following a winter cold. His care provider would recognize that which of the following microorganisms and treatments is most likely to be effective? A) Respiratory syncytial virus treated with intubation B) Parainfluenza virus treated with a mist tent and oxygen therapy C) Haemophilus influenza treated with appropriate antibiotics D) Staphylococcus aureus treated with bronchodilators and mist tent

B

A 34-year-old man who is an intravenous drug user has presented to the emergency department with malaise, abdominal pain, and lethargy. The health care team wants to rule out endocarditis as a diagnosis. Staff of the department would most realistically anticipate which of the following sets of diagnostics? A) CT of the heart, chest x-ray, and ECG B) Echocardiogram, blood cultures, and temperature C) ECG, blood pressure, and stress test D) Cardiac catheterization, chest x-ray, electrolyte measurement, and white cell count

B

A 41-year-old male client has presented to the emergency department with an acute onset of increased respiratory rate and difficulty breathing. STAT chest x-ray indicates diffuse bilateral infiltrates of his lung tissue, and ECG displays no cardiac dysfunction. What is this client's most likely diagnosis? A) Cor pulmonale B) Acute lung injury C) Pulmonary hypertension D) Sarcoidosis

B

A 44-year-old female patient presents to the emergency department with abnormal bleeding and abdominal pain that is later attributed to gallbladder disease. Which of the following diagnoses would the medical team be most justified in suspecting as a cause of the patient's bleeding? A) Excess calcium B) Vitamin K deficiency C) Hemophilia B D) Idiopathic immune thrombocytopenic purpura (ITP)

B

A 51-year-old female client who is 2 days postoperative in a surgical unit of a hospital is at risk of developing atelectasis as a result of being largely immobile. Which of the following teaching points by her nurse is most appropriate? A) "Being in bed increases the risk of fluid accumulating between your lungs and their lining, so it's important for you to change positions often." B) "You should breathe deeply and cough to help your lungs expand as much as possible while you're in bed." C) "Make sure that you stay hydrated and walk as soon as possible to avoid us having to insert a chest tube." D) "I'll proscribe bronchodilator medications that will help open up your airways and allow more oxygen in."

B

A 55-year-old male who is beginning to take a statin drug for his hypercholesterolemia is discussing cholesterol and its role in health and illness with his physician. Which of the following aspects of hyperlipidemia would the physician most likely take into account when teaching the patient? A) Hyperlipidemia is a consequence of diet and lifestyle rather than genetics. B) HDL cholesterol is often characterized as being beneficial to health. C) Cholesterol is a metabolic waste product that the liver is responsible for clearing. D) The goal of medical treatment is to eliminate cholesterol from the vascular system.

B

A 62-year-old female smoker is distraught at her recent diagnosis of small cell lung cancer (SCLC). How can her physician most appropriately respond to her? A) "I'm sure this is very hard news to hear, but be aware that with aggressive treatment, your chances of beating this are quite good." B) "This is very difficult to hear, I'm sure, and we have to observe to see if it spreads because that often happens." C) "I'm very sorry to have to give you this news; I'd like to talk to you about surgical options, however." D) "This is a difficult diagnosis to receive, but there is a chance that the cancer may go into remission."

B

A 66-year-old female patient has presented to the emergency department because of several months of intermittently bloody stool that has recently become worse. The woman has since been diagnosed with a gastrointestinal bleed secondary to overuse of nonsteroidal anti-inflammatory drugs that she takes for her arthritis. The health care team would realize that which of the following situations is most likely to occur? A) The woman has depleted blood volume due to her ongoing blood loss. B) She will have iron deficiency anemia due to depletion of iron stores. C) The patient will be at risk for cardiovascular collapse or shock. D) She will have delayed reticulocyte release.

B

A 77-year-old patient with a history of coronary artery disease and heart failure has arrived in the emergency room with a rapid heart rate and feeling of "impending doom." Based on pathophysiologic principles, the nurse knows the rapid heart rate could A) decrease renal perfusion and result in the development of ascites. B) be a result of catecholamines released from SNS that could increase the myocardial oxygen demand. C) desensitize the -adrenergic receptors leading to increase in norepinephrine levels. D) prolong the electrical firing from the SA node resulting in the development of a heart block.

B

A client has been diagnosed with mitral valve stenosis following his recovery from rheumatic fever. Which of the following teaching points would be most accurate to convey to the client? A) "The normal tissue that makes up the valve between the right sides of your heart has stiffened." B) "Your mitral valve isn't opening up enough for blood to flow into the part of your heart that sends blood into circulation." C) "Your heart's mitral valve isn't closing properly so blood is flowing backward in your heart and eventually into your lungs." D) "The valve between your left ventricle and left atria is infected and isn't allowing enough blood through."

B

A client with a newborn infant is also the caregiver for her 75-year-old mother, who lives with them and who has diabetes. The client requests pneumonia vaccinations for her entire household. Which vaccine is most likely to be effective for the baby? A) Since the baby's immune system is mature at birth, regular vaccine is appropriate. B) There is no effective vaccine for newborn infants. C) The 23-valent vaccine will be effective. D) No vaccine is necessary for the baby if the nursing mother is immunized

B

A group of novice nursing students are learning how to manually measure a client's blood pressure using a stethoscope and sphygmomanometer. Which of the following statements by students would the instructor most likely need to correct? A) "I'll inflate the cuff around 30 mm Hg above the point at which I can't palpate the client's pulse." B) "If my client's arm is too big for the cuff, I'm going to get a BP reading that's artificially low." C) "The accuracy of the whole process depends on my ability to clearly hear the Korotkoff sounds with the bell of my stethoscope." D) "With practice, my measurement of clients' blood pressures with this method will be more accurate than with automated machines."

B

A hospital client is receiving intravenous infusion of heparin for treatment of a pulmonary embolus. Which of the following phenomena is most likely to occur, resulting in the drug's therapeutic effect? A) Inhibition of vitamin K synthesis in the liver B) Suppression of fibrin formation C) Deactivation of the intrinsic clotting pathway D) Inhibition of ADP-induced platelet aggregation

B

A hospital laboratory technologist is analyzing the complete blood count (CBC) of a patient. Which of the following statements best reflects an aspect of the platelets that would constitute part of the CBC? A) Platelets originate with granulocyte colony-forming units (CFU). B) The half-life of a platelet is typically around 8 to 12 days. C) The -granules of platelets contribute primarily to vasoconstriction. D) New platelets are released from the bone marrow into circulation

B

A medical student is working with a 61-year-old male client in the hospital who has presented with a new onset of atrial fibrillation. Which of the following courses of treatment will the student most likely expect the attending physician to initiate? A) Immediate cardioversion followed by surgery to correct the atrial defect B) Anticoagulants and beta-blockers to control rate C) Antihypertensives and constant cardiac monitoring in a high acuity unit D) Diuretics, total bed rest, and cardioversion if necessary

B

A number of patients in an acute cardiac care unit of a hospital have diagnoses of impaired cardiac conduction. Which of the following patients is most deserving of immediate medical attention? A) A 46-year-old man whose cardiac telemetry shows him to be in ventricular tachycardia B) A 69-year-old woman who has entered ventricular fibrillation C) A 60-year-old man with premature ventricular contractions (PVC) and a history of atrial fibrillation D) A 60-year-old woman who has just been diagnosed with a first-degree AV block

B

A nurse is administering morning medications to a number of patients on a medical unit. Which of the following medication regimens is most suggestive that the patient has a diagnosis of heart failure? A) Antihypertensive, diuretic, antiplatelet aggregator B) Diuretic, ACE inhibitor, beta-blocker C) Anticoagulant, antihypertensive, calcium supplement D) Beta-blocker, potassium supplement, anticoagulant

B

A nurse is providing care for an older, previously healthy adult male who has been diagnosed today with pneumococcal pneumonia. Which of the following signs and symptoms is the nurse most likely to encounter? A) The man will be hypotensive and febrile and may manifest cognitive changes. B) The patient will have a cough producing clear sputum, and he will have faint breath sounds and fine crackles. C) The patient will have copious bloody sputum and diffuse chest pain and may lose his cough reflex. D) The patient will lack lung consolidation and will have little, if any, sputum production.

B

A nurse who works in a neonatal intensive care unit is providing care for an infant born at 26 weeks' gestation. Which of the following assessments would lead the nurse to suspect that the infant has developed respiratory distress syndrome (RDS)? A) The infant's blood pressure and temperature are normal measurements as expected. B) Infant is grunting and has notable intercostal retractions with respirations. C) Infant has poor motor skills and limited limb range of motion. D) Infant has apnea lasting 5 to 10 seconds with a decrease in heart rate, which reverses with tactile stimulation.

B

A patient asks the health care provider why his lower legs look purple. The health care provider will base her response on which pathophysiological principle? A) Too much trauma breaks capillaries, and they bleed into the tissue. B) The bruising around the ankles is due to the fact that it is a dependent area where the capillary pressure is higher. C) There is a problem with his plasminogen levels. D) Morbid obesity causes veins to enlarge and bleed into tissues due to stress the abdomen is placing on the vascular system.

B

A patient in the intensive care unit has a blood pressure of 87/39 and has warm, flushed skin accompanying his sudden decline in level of consciousness. The patient also has arterial and venous dilatation and a decrease in systemic vascular resistance. What is this client's most likely diagnosis? A) Hypovolemic shock B) Septic shock C) Neurogenic shock D) Obstructive shock

B

A patient is admitted for a relapse for sarcoidosis. Knowing this is usually caused by an inflammatory process, the nurse can anticipate administering A) a bronchodilator. B) a corticosteroid. C) aspirin. D) an albuterol inhaler

B

A patient is admitted to the outpatient diagnostic unit for further testing to identify the cause of the uncontrolled secondary hypertension. In preparation, the nurse should anticipate that which of the following diagnostic procedures will provide the most definitive diagnosis? A) Routine ultrasound of kidney B) Renal arteriography C) Echocardiography D) Serum creatinine level

B

A patient with a new automatic implantable cardioverter-defibrillator (AICD) asks the nurse what happens if he goes into that deadly heart rhythm again. The nurse will base her response knowing that the AICD will A) periodically fire just to test for lead placement and battery life. B) respond to ventricular tachyarrhythmia by delivering a shock within 10 to 20 seconds of its onset. C) use radiofrequency energy to deliver an electrical shock through the site where the lethal rhythm originates. D) remove scar tissue and aneurysm during placement of electrodes and then will shock if paradoxical ventricular movement is located.

B

A physician has ordered the measurement of a cardiac patient's electrolyte levels as part of the client's morning blood work. Which of the following statements best captures the importance of potassium in the normal electrical function of the patient's heart? A) Potassium catalyzes the metabolism of ATP, producing the gradient that results in electrical stimulation. B) Potassium is central to establishing and maintaining the resting membrane potential of cardiac muscle cells. C) The impermeability of cardiac cell membranes to potassium allows for action potentials achieved by the flow of sodium ions. D) The reciprocal movement of one potassium ion for one sodium ion across the cell membrane results in the production of an action potential.

B

A pneumonia that occurs 48 hours or more after admission to the hospital is considered A) community-acquired pneumonia. B) hospital-acquired pneumonia. C) viral pneumonia. D) immunocompromised pneumonia.

B

A surgeon is explaining to the parents of a 6-year-old boy the rationale for the suggestion of removing the boy's spleen. Which of the following teaching points would be most accurate? A) "Ferritin is the activated and usable form of iron that your red blood cells can use to transport oxygen." B) "Ferritin is a stored form of iron that indirectly shows me whether you would benefit from iron pills." C) "Ferritin is a protein-iron complex that allows your red blood cells to make use of the iron that you consume in your diet." D) "Ferritin is the form of iron that is transported in your blood plasma to red blood cells that need it."

B

An ECG technician is placing leads on a patient who has presented to the emergency department with a sudden onset of chest pain. The technician would recognize which of the following facts about the placement of leads and the achievement of a clinically accurate ECG? A) The electrical potential recorded by a lead on an extremity will vary significantly depending on where the lead is placed on the extremity. B) The chest leads measure electrical activity on the horizontal plane, while limb leads measure it on the vertical plane. C) Limb leads measure the electrical activity of the heart indirectly through the activity of adjacent skeletal muscle. D) A total of 12 chest leads are necessary to attain the most accurate ECG.

B

An elderly patient arrives to the health care provider's office complaining of a "sore" that would not heal on his lower leg. Upon assessment, the nurse finds thin, shiny, bluish brown pigmented desquamative skin. It is located medially over the lower leg. The nurse will educate the patient that the usual treatment is A) hydrotherapy to facilitate improvement in circulation. B) compression therapy to help facilitate blood flow back to the vena cava. C) initiation of Coumadin therapy to maintain an INR of 2 to 3 above norm. D) long-term antibiotic therapy to facilitate healing of the wound.

B

During a routine physical examination of a 66-year-old woman, her nurse practitioner notes a pulsating abdominal mass and refers the woman for further treatment. The nurse practitioner is explaining the diagnosis to the client, who is unfamiliar with aneurysms. Which of the following aspects of the pathophysiology of aneurysms would underlie the explanation the nurse provides? A) Aneurysms are commonly a result of poorly controlled diabetes mellitus. B) Hypertension is a frequent modifiable contributor to aneurysms. C) Individuals with an aneurysm are normally asymptomatic until the aneurysm ruptures. D) Aneurysms can normally be resolved with lifestyle and diet modifications.

B

In the grocery store, a nurse overhears a teenage mother intentionally shaming and verbally reprimanding a child in public. The mother also grabbed the child's stuffed animal and tore the limbs off. From what the nurse remembers about abuse, this would be classified as a form of A) physical abuse. B) emotional abuse. C) sexual abuse. D) neglect.

B

In which of the following patient situations would a physician be most justified in preliminarily ruling out pericarditis as a contributing pathology to the patient's health problems? A) A 61-year-old man whose ECG was characterized by widespread T-wave inversions on admission but whose T waves have recently normalized B) A 77-year-old with diminished S3 and S4 heart tones, irregular heart rate, and a history of atrial fibrillation C) A 56-year-old obese man who is complaining of chest pain that is exacerbated by deep inspiration and is radiating to his neck and scapular ridge D) A 60-year-old woman whose admission blood work indicates elevated white cells, erythrocyte sedimentation rate, and C-reactive protein levels

B

The family members of an elderly patient are wondering why his "blood counts" are not rising after his last GI bleed. They state, "He has always bounced back after one of these episodes, but this time it isn't happening. Do you know why?" The nurse will respond based on which of the following pathophysiological principles? A) "Everything slows down when you get older. You just have to wait and see what happens." B) "Due to stress, the red blood cells of older adults are not replaced as promptly as younger people." C) "The doctor may start looking for another cause of his anemia, maybe cancer of the bone." D) "Don't worry about it. We can always give him more blood."

B

The nurse working in the ICU knows that chronic elevation of left ventricular end-diastolic pressure will result in the patient displaying which of the following clinical manifestations? A) Chest pain and intermittent ventricular tachycardia B) Dyspnea and crackles in bilateral lung bases C) Petechia and spontaneous bleeding D) Muscle cramping and cyanosis in the feet

B

What topic should health promotion initiatives emphasize if the target audience is parents of preschoolers and the goal is to minimize mortality? A) Handwashing as an infection control measure B) Injury prevention especially when the child is near water C) Identifying signs of child abuse and neglect D) The importance of good nutrition

B

When advising a morbidly obese patient about the benefits of weight reduction, which of the following statements would be most accurate to share? A) "All you need to do is stop drinking sodas and sugary drinks." B) "A 10 lb loss of weight can produce a decrease in blood pressure." C) "An increased 'waist-to-hip' ratio can lead to too much pressure on the liver and intestines." D) "If your leptin (hormone) level is too low, you are at increased risk for developing high BP."

B

When trying to educate a patient about the release of free radicals and the role they play in formation of atherosclerosis, which of the following statements is most accurate? A) The end result of oxidation is rupture of the plaque resulting in hemorrhage. B) Activated cells that release free radicals oxidize LDL, which is harmful to the lining of your blood vessels. C) Oxidized free radicals produce toxic metabolic waste that can kill liver cells. D) Activated cells roam in the vascular system looking for inflammatory cells to engulf.

B

Which of the following assessment findings of a newly admitted 30-year-old male client would be most likely to cause his physician to suspect polyarteritis nodosa? A) The man's blood work indicates polycythemia (elevated red cell levels) and leukocytosis (elevated white cells). B) The man's blood pressure is 178/102, and he has abnormal liver function tests. C) The man is acutely short of breath, and his oxygen saturation is 87%. D) The man's temperature is 101.9°F, and he is diaphoretic (heavily sweating).

B

Which of the following patients should the nurse be assessing for long QT syndrome? A) A 95-year-old patient with Alzheimer's who is having periods of apnea B) A 32-year-old male admitted for cocaine overdose with long history of illicit drug abuse C) A 56-year-old female admitted for total hysterectomy due to excessive bleeding and clotting D) A 68-year-old male who was in a car accident with sternal bruising and fractured femur

B

Which of the following phenomena is most likely occurring during a child's alveolar stage of lung development? A) Terminal alveolar sacs are developing, and surfactant production is beginning. B) A single capillary network exists, and the lungs are capable of respiration. C) The conducting airways are formed, but respiration is not yet possible. D) Primitive alveoli are formed, and the bronchi and bronchioles become much larger.

B

Which of the following teaching points would be most appropriate for a group of older adults who are concerned about their cardiac health? A) "People with plaque in their arteries experience attacks of blood flow disruption at seemingly random times." B) "The plaque that builds up in your heart vessels obstructs the normal flow of blood and can even break loose and lodge itself in a vessel." C) "Infections of any sort are often a signal that plaque disruption is in danger of occurring." D) "The impaired function of the lungs that accompanies pneumonia or chronic obstructive pulmonary disease is a precursor to plaque disruption."

B

While being on subcutaneous heparin injections for deep vein thrombosis during her latter pregnancy, a patient begins to experience major side effects. Her OB-GYN physician has called in a specialist who thinks that the patient is experiencing heparin-induced thrombocytopenia. The nurse should anticipate which of the following orders? A) Decrease the dose of heparin from 5000 units b.i.d to 3000 units b.i.d B) Immediately discontinue the heparin therapy C) Switch to Coumadin 2.5 mg once/day D) Infuse FFP stat

B

Which of the following ECG patterns would the nurse observe in a patient admitted for arrhythmogenic right ventricular cardiomyopathy/dysplasia (ARVC/D)? Select all that apply. A) Atrial flutter B) Ventricular tachycardia with left bundle branch block pattern C) T-wave inversion in the right precordial leads D) Sinus arrhythmia with a first-degree AV block E) Development of a "U" wave following a normal T wave

B C

The nursery has just admitted a new infant born 1 hour ago. While performing an assessment, the nurse suspects the infant may have hypoglycemia based on which of the following assessment data? Select all that apply. A) Heel stick glucose value of 50 mg/dL B) Infant having periods of apnea requiring physical stimulation C) Muscle twitching noted while lying in crib undisturbed by nurses D) Hyperactive reflexes noted especially when crying E) Poor suck reflex resulting in an inability to feed properly

B C E

A 14-year-old boy has experienced a pronounced growth spurt over the last several months. While discussing this with his parents, the nurse educates what normal male growth patterns contain. Of the following, which are accurate statements to relay to the parents? Select all that apply. A) Most males will complete their growth spurt by age 16. B) It is not usual for their son to gain up to 30 kg in weight. C) With parathyroid hormone involvement, your son may be at risk for fractures. D) Expect the thorax to become broader and for the pelvis to remain narrow. E) Some children have stunted growth in their arms or legs.

B D

As a consequence of a long-standing lung disease, a client is in a chronic state of hypoxia. Which of the following phenomena would the client's care team be most justified in anticipating? Select all that apply. A) Metabolic alkalosis B) Increased erythropoietin production C) Pulmonary vasodilation D) Hyperventilation E) Personality changes

B D E

Upon admission to the ICU, a patient with a history of cor pulmonale will likely be exhibiting which of the following clinical manifestations of right heart failure? Select all that apply. A) Fine crackles throughout both lung fields B) +4 pitting edema in lower extremities C) Expectorating copious amounts of frothy, pink sputum D) Jugular vein distension E) Altered level of consciousness

B D E

A 10-year-old boy has a body mass index that places him in the 96th percentile for his age and gender. While educating the parents about obesity, the nurse should emphasize that his weight may predispose him to the development of A) scoliosis. B) respiratory infections. C) gastrointestinal disorders. D) type 2 diabetes.

D

A 22-year-old male is experiencing hypovolemic shock following a fight in which his carotid artery was cut with a broken bottle. What immediate treatments are likely to most benefit the man? A) Resolution of compensatory pulmonary edema and heart arrhythmias B) Infusion of vasodilators to foster perfusion and inotropes to improve heart contractility C) Infusion of normal saline or Ringer lactate to maintain the vascular space D) Administration of oxygen and epinephrine to promote perfusion

C

A 3-year-old child with right-sided heart failure has been admitted for worsening of his condition. Which of the following assessments would be considered one of the earliest signs of systemic venous congestion in this toddler? A) Breathlessness with activity B) Excessive crying C) Enlargement of the liver D) Increased urine output

C

A 30-year-old woman presents at a hospital after fainting at a memorial service, and she is diagnosed as being in neurogenic shock. Which of the following signs and symptoms is she most likely to display? A) Faster than normal heart rate B) Pain C) Dry and warm skin D) Increased thirst

C

A 44-year-old Caucasian woman is being treated in an airport infirmary after she developed a painful, swollen leg during a transatlantic flight in economy class. The woman is suspected of having deep vein thrombosis (DVT) and is questioning the paramedics about why this might be the case, given that she has twice previously had similar experiences. Which of the following teaching points by the airport medical staff would be most appropriate? A) "A lot of Caucasian people have a genetic mutation that causes platelets to stick to their blood vessel walls." B) "There is a genetic disorder that causes many Caucasians to form more clots in their blood vessels." C) "A lot of Caucasians have an inherited inability to dissolve clots that form in their bodies." D) "Your doctor might be able to tell you if you've inherited a predisposition to bleeding in your veins."

C

A 62-year-old female with a diagnosis of acute and chronic renal failure secondary to diabetes mellitus is receiving her weekly injection of epoetin, a supplementary form of erythropoietin. Which of the following statements best captures the necessity of this medication? A) Erythropoietin is needed in order for stem cells to proliferate into committed erythroid precursors. B) Erythropoietin is necessary for the accurate sensation of hypoxia that stimulates erythropoiesis. C) Erythropoietin causes the erythrocyte colony-forming units to proliferate and mature. D) Erythropoietin facilitates the extrusion of the reticulocyte nucleus and the formation of true erythrocytes

C

A 66-year-old male presents to the emergency room accompanied by his wife who claims that he has been acting confused. The man is complaining of a sudden onset of severe weakness and malaise and has a dry cough and diarrhea. His temperature is 102.8°F, and his blood work indicates his sodium level at 126 mEq/L (normal 135 to 145 mEq/L). Based on this assessment, the nurse suspects the patient has A) bronchopneumonia. B) Mycoplasma pneumonia. C) Legionella pneumonia. D) pneumococcal pneumonia.

C

A 70-year-old male client presents to the emergency department complaining of pain in his calf that is exacerbated when he walks. His pedal and popliteal pulses are faintly palpable, and his leg distal to the pain is noticeably reddened. The nurse knows that the client is likely experiencing which of the following medical diagnosis/possible treatment plans listed below? A) Acute arterial occlusion that will be treated with angioplasty B) Raynaud disease that will require antiplatelet medications C) Atherosclerotic occlusive disease necessitating thrombolytic therapy D) Giant cell temporal arteritis that will be treated with corticosteroids

C

A 71-year-old man is slated for pacemaker insertion for treatment of a third-degree AV block. The man's nurse has been educating him about his diagnosis and treatment and answering the numerous questions he has about his health problem. Which of the following teaching points should the nurse include in this patient teaching? A) "This is almost certainly a condition that you were actually born with, but that is just now becoming a serious problem." B) "Because the normal electrical communication in lacking, the bottom parts of your heart are beating especially fast to compensate for inefficiency." C) "The root problem is that the top chambers of your heart and the bottom chambers of your heart aren't coordinating to pump blood efficiently." D) "If left untreated, this would have put you at great risk of stroke or heart attack."

C

A 72-year-old woman with a recent onset of syncopal episodes has been referred to a cardiologist by her family physician. As part of the client's diagnostic workup, the cardiologist has ordered her to wear a Holter monitor for 24 hours. Which of the following statements best captures an aspect of Holter monitoring? A) A Holter monitor is preferable to standard ECG due to its increased sensitivity to cardiac electrical activity. B) The primary goal is to allow the cardiologist to accurately diagnose cardiomyopathies. C) Accurate interpretation of the results requires correlating the findings with activity that the woman was doing at the time of recording. D) Holter monitors are normally set to record electrical activity of the heart at least once per hour.

C

A 74-year-old man is being assessed by a nurse as part of a weekly, basic health assessment at the long-term care facility where he resides. His blood pressure at the time is 148/97 mm Hg, with a consequent pulse pressure of 51 mm Hg. The nurse would recognize that which of the following is the most significant determinant of the resident's pulse pressure? A) Blood volume, resistance, and flow B) The cardiac reserve or possible increase in cardiac output over normal resting level C) The amount of blood that his heart ejects from the left ventricle during each beat D) The relationship between total blood volume and resting heart rate

C

A 78-year-old man has been experiencing nocturnal chest pain over the last several months, and his family physician has diagnosed him with variant angina. Which of the following teaching points should the physician include in his explanation of the man's new diagnosis? A) "I'll be able to help track the course of your angina through regular blood work that we will schedule at a lab in the community." B) "With some simple lifestyle modifications and taking your heparin regularly, we can realistically cure you of this." C) "I'm going to start you on low-dose aspirin, and it will help greatly if you can lose weight and keep exercising." D) "There are things you can do to reduce the chance that you will need a heart bypass, including limiting physical activity as much as possible."

C

A child has been diagnosed with thalassemia. Which of the following comorbidities may occur as a result of having thalassemia? A) Hypocoagulation B) Iron deficiency C) Splenomegaly D) Neutropeniabb

C

A child with rhinosinusitis should be monitored for complications. Which of the following assessment findings would alert the nurse that a complication is developing? A) Purulent nasal discharge B) Temperature of 100.8°F C) Periorbital edema D) Complaints of headache

C

A formerly normotensive woman, pregnant for the first time, develops hypertension and headaches at 26 weeks' gestation. Her blood pressure is 154/110 mm Hg, and she has proteinuria. What other lab tests should be ordered for her? A) Plasma angiotensin I and II and renin B) Urinary sodium and potassium C) Platelet count, serum creatinine, and liver enzymes D) Urinary catecholamines and metabolites

C

A health educator is performing a health promotion workshop with the staff of a large, urban homeless shelter, and a component of the teaching centers around tuberculosis. One of the staff members comments, "Anyone who's had contact with tuberculosis in the past can give it to any of the other residents of the shelter, even if they didn't get sick themselves." How could the educator best respond to this comment? A) "Many people do manage to fight off the infection, but you're right: they can still spread it by coughing or sneezing." B) "If someone has been previously exposed to tuberculosis, they are particularly infectious because they are often unaware of the disease." C) "Actually, people who have the latent form of the disease won't be sick and can't spread it either." D) "There isn't any real risk of them spreading it, but we would like to vaccinate everyone who's had any contact with it in the past."

C

A number of older adults have come to attend a wellness clinic that includes both blood pressure monitoring and education about how to best control blood pressure. Which of the leader's following teaching points is most accurate? A) "It's important to minimize the amount of potassium and, especially, sodium in your diet." B) "High blood pressure is largely controllable, except for those with a significant family history or African Americans." C) "Too much alcohol, too little exercise, and too much body fat all contribute to high blood pressure." D) "Hypertension puts you at a significant risk of developing type 2 diabetes later in life."

C

A nurse educator is teaching a group of nurses at a long-term care facility about atrial fibrillation in light of its prevalence in older adults. Which of the following statements by the nurses would the educator most want to correct? A) "The electrical impulses go in chaotic directions, and so the atria can't contract properly." B) "An ECG of someone in atrial fibrillation would be almost random in appearance." C) "The contraction of the ventricles and the atria can range from 400 to 600 beats/minute." D) "It can be hard to measure at the bedside because not all ventricular beats make a palpable pulse."

C

A nurse is performing a 5-minute Apgar score on a newborn female. Which of the following characteristics of the infant's current condition would not be reflected in the child's Apgar score? A) The baby's heart rate is 122 beats/minute. B) The infant displays a startle reflex when the crib is accidentally kicked. C) The child's temperature is 35.0°C (95°F) by axilla. D) The infant's skin is pink in color.

C

A nurse is providing care for a number of older clients on a restorative care unit of a hospital. Many of the clients have diagnoses or histories of hypertension, and the nurse is responsible for administering a number of medications relevant to blood pressure control. Which of the following assessments would the nurse be most justified in eliminating during a busy morning on the unit? A) Checking the recent potassium levels of a client receiving an ACE inhibitor B) Measuring the heart rate of a client who takes a beta-adrenergic blocker C) Measuring the pulse of a client taking an ACE inhibitor D) Noting the sodium and potassium levels of a client who is receiving a diuretic

C

A nurse will be providing care for a female patient who has a diagnosis of heart failure that has been characterized as being primarily right sided. Which of the following statements best describes the presentation that the nurse should anticipate? The client A) has a distended bladder, facial edema, and nighttime difficulty breathing. B) complains of dyspnea and has adventitious breath sounds on auscultation (listening). C) has pitting edema to the ankles and feet bilaterally, decreased activity tolerance, and occasional upper right quadrant pain. D) has cyanotic lips and extremities, low urine output, and low blood pressure

C

A physical assessment of a 28-year-old female patient indicates that her blood pressure in her legs is lower than that in her arms and that her brachial pulse is weaker in her left arm than in her right. In addition, her femoral pulses are weak bilaterally. Which of the following possibilities would her care provider most likelysuspect? A) Pheochromocytoma B) Essential hypertension C) Coarctation of the aorta D) An adrenocortical disorder

C

A short, nonsmoking 44-year-old male presents to the emergency room with left-sided chest pain and a cough. He states that the pain started abruptly and worsens with deep breathing and coughing. He denies recent injury. Assessment includes shallow respirations with a rate of 36, normal breath sounds, and no cyanosis. Which condition is most likely causing his symptoms? A) Myocardial infarction B) Spontaneous pneumothorax C) Pleuritis related to infection D) Obstructive atelectasis

C

A surgeon is explaining to the parents of a 6-year-old boy the rationale for the suggestion of removing the boy's spleen. Which of the following teaching points would be most accurate? A) "We believe that your son's spleen is causing the destruction of many of his blood platelets, putting him at a bleeding risk." B) "Your son's spleen is inappropriately filtering out the platelets from his blood and keeping them from normal circulation." C) "Your son's spleen is holding on to too many of his platelets, so they're not available for clotting." D) "We think that his spleen is inhibiting the production of platelets by his bone marrow."

C

Assuming that they have not responded to drug therapy, which of the following clients is likely to be the best candidate for surgical cardiac ablation? A) A 62-year-old woman with peripheral vascular disease who has experienced multiple episodes of torsade des pointes B) A 75-year-old man with diabetes but no previous heart disease that suddenly develops syncope due to sick sinus syndrome C) A 46-year-old man with unstable angina and a history of myocardial infarction who is found to have long QT syndrome and episodes of frequent ventricular arrhythmias D) A 22-year-old woman with an atrial septal defect who has recurrent paroxysmal atrial flutter with rapid ventricular rate associated with her caffeine intake.

C

Due to rapid neural growth, a child can begin to control the bowel and bladder sphincters by what age? A) 12 months B) 18 months C) 2 years D) 4 years

C

During a prenatal education class, an expectant mother tells the group about a friend whose blood pressure became so high during pregnancy that she had to be admitted to hospital. Which of the following statements should the nurse include in response to this? A) "A large increase in blood pressure is a normal part of the changes in blood circulation that accompany pregnancy." B) "By avoiding salt, staying active, and minimizing weight gain, you can prevent this during your pregnancy." C) "Essentially, experts don't really know why so many pregnant women develop high blood pressure." D) "I'm sure this was hard for your friend, but rest assured that it won't affect your baby even if it affects you."

C

Four patients were admitted to the emergency department with severe chest pain. All were given preliminary treatment with aspirin, morphine, oxygen, and nitrates and were monitored by ECG. Which patient most likely experienced myocardial infarction? A) A 33-year-old male whose pain started at 7 AM during moderate exercise and was relieved by nitrates; ECG was normal; cardiac markers remained stable. B) A 67-year-old female whose pain started at 2 AM while she was asleep and responded to nitrates; the ECG showed arrhythmias and ST-segment elevation; cardiac markers remained stable. C) An 80-year-old woman whose pain started at 6 AM shortly after awakening and was not relieved by nitrates or rest; the ECG showed ST-segment elevation with inverted T waves and abnormal Q waves; levels of cardiac markers subsequently rose. D) A 61-year-old man whose pain started at 9 AM during a short walk and responded to nitrates, but not to rest; ECG and cardiac markers remained stable, but anginal pattern worsened.

C

If a virus has caused inflammation resulting in endothelial dysfunction, an excessive amount of endothelins in the blood can result in A) arterial wall weakening resulting in aneurysm formation. B) release of excess fatty plaque causing numerous pulmonary emboli. C) contraction of the underlying smooth muscles within the vessels. D) overproduction of growth factors resulting in new vessel production.

C

If an Rh-negative mother is giving birth to an Rh-positive infant, the nurse should be prepared to administer A) antihistamines like Benadryl. B) alpha interferon. C) Rh immune globulin. D) a monoclonal antibody like infliximab

C

In the early morning, an African American woman brings her 5-year-old son to the emergency room. The boy is wheezing, is short of breath, and has a dry cough. The mother states that he has always been very healthy. He went to bed with only a slight cold and a runny nose but woke her with his coughing shortly after 4 AM. His symptoms worsened so dramatically that she brought him to the hospital. The care team would most likely suspect that he has A) respiratory syncytial virus. B) influenza. C) asthma. D) pneumonia.

C

In which of the following hospital patients would the care team most realistically anticipate finding normal cholesterol levels? A) A 44-year-old male admitted for hyperglycemia and with a history of diabetic neuropathy B) A 77-year-old female admitted for rheumatoid arthritis exacerbation who is receiving hormone replacement therapy and with a history of hypothyroidism C) A 51-year-old male with a diagnosis of hemorrhagic stroke and consequent unilateral weakness D) A morbidly obese 50-year-old female who is taking diuretics and a beta-blocker to treat her hypertension

C

In which of the following patients is the emergency department staff most likely to suspect an abdominal aortic aneurysm? A) A 60-year-old client with diminished oxygen saturation, low red blood cell levels, and pallor B) A 70-year-old woman with jugular venous distention, shortness of breath, and pulmonary edema C) A 66-year-old client with facial edema, cough, and neck vein distention D) An 81-year-old man with acute cognitive changes as well as difficulty in speaking and swallowing

C

New parents were just told by their physician that their son is two standard deviations above the mean. The parents later asked the nurse what that means. The nurse will explain by stating, A) "If your child is one standard deviation from the norm that translates to mean, he will be taller than 50% of his peers." B) "This is great news since it means you will have a larger baby than most." C) "Being two standard deviations above the mean translates into that your child will likely be taller than 95% of children in the population." D) "With the mean being average at 50%, two standard deviations means that your child will be at least 99.7% taller than his brother."

C

On a routine physical exam visit, the physician mentions that he hears a new murmur. The patient gets worried and asks, "What does this mean?" The physician responds, A) "It would be caused by stress. Let's keep our eye on it and see if it goes away with your next visit." B) "This could be caused by an infection. Have you been feeling well the past few weeks?" C) "One of your heart valves is not opening properly. We need to do an echocardiogram to see which valve is having problem." D) "This may make you a little more fatigued than usual. Let me know if you start getting dizzy or light-headed."

C

The cardiologist just informed a patient that he has a reentry circuit in the electrical conduction system in his heart. This arrhythmia is called Wolff-Parkinson-White (WPW) syndrome. After the physician has left the room, the patient asks the nurse to explain this to him. Which of the following statements most accurately describes what is happening? A) "This means that the SA node (which is the beginning of your heart's electrical system) has been damaged and is no longer functioning normal." B) "You must have a large clot in one of your arteries that supply oxygenated blood to the special conduction cells in your heart." C) "There is an extra, abnormal electrical pathway in the heart that leads to impulses traveling around the heart very quickly, in a circular pattern, causing the heart to beat too fast." D) "For some reason, your electrical system is not on full charge, so they will have to put in new leads and a pacemaker to make it work better."

C

The initial medical management for a symptomatic patient with obstructive hypertrophic cardiomyopathy (HCM) would be administering a medication to block the effects of catecholamines. The nurse will anticipate administering which of the following medications? A) Lisinopril, an ACE inhibitor B) Lasix, a diuretic C) Propranolol, a -adrenergic blocker D) Lanoxin, an inotropic

C

The neonatal ICU nurse is aware that type II alveolar cells produce surfactant, and they usually develop at how many weeks of gestation? A) 17 to 18 weeks B) 19 to 20 weeks C) 24 to 28 weeks D) 34 to 38 weeks

C

Two nursing students are attempting to differentiate between the presentations of immune thrombocytopenic purpura (ITP) and thrombotic thrombocytopenic purpura (TTP). Which of the students' following statements best captures an aspect of the two health problems? A) "Both diseases can result from inadequate production of thrombopoietin by megakaryocytes." B) "ITP can be either inherited or acquired, and if it's acquired, it involves an enzyme deficiency." C) "Both of them involve low platelet counts, but in TTP, there can be more, not less, hemostasis. D) "TTP can be treated with plasmapheresis, but ITP is best addressed with transfusion of fresh frozen plasma."

C

When educating a student who lives in a crowded apartment and diagnosed with tuberculosis, the college school nurse will emphasize, A) "Once your fever goes away, you can stop taking the streptomycin injection." B) "If isoniazid makes you nauseous, we can substitute something milder." C) "To destroy this bacterium, you must strictly adhere to a long-term drug regimen." D) "You will have to wear an N95 mask while on campus at all times."

C

Which of the following assessment findings of a male infant 14 hours postpartum would be considered abnormal and would require further assessment and possible intervention? A) The baby's first stool appears to contain blood. B) The child is unable to breathe through his mouth. C) The baby's skin has a yellowish orange hue. D) The child's suck is weak when placed at his mother's breast.

C

Which of the following clinical findings would be most closely associated with a client who has interstitial lung disease in comparison to chronic obstructive pulmonary disease (COPD)? A) Audible wheezing on expiration B) Reduced expiratory flow rates C) Decreased tidal volume D) Normal forced expiratory volume

C

Which of the following patients will likely experience difficulty in maintaining lipoprotein synthesis resulting in elevated LDL levels? A) A 55-year-old male admitted for exacerbation of chronic obstructive pulmonary disease (COPD) B) A 44-year-old female admitted for hysterectomy due to cervical cancer with metastasis C) A 35-year-old patient with a history of hepatitis C and B with end-stage liver disease D) A 27-year-old patient with pancreatitis related to alcohol abuse

C

Which of the following phenomena would be most likely to accompany increased myocardial oxygen demand (MVO2)? A) Inadequate ventricular end-diastolic pressure B) Use of calcium channel blocker medications C) Increased aortic pressure D) Ventricular atrophy

C

Which of the following residents of a long-term care facility is most likely to be exhibiting the signs and symptoms of chronic obstructive pulmonary disease (COPD)? A) A 79-year-old lifetime smoker who is complaining of shortness of breath and pain on deep inspiration B) An 81-year-old smoker who has increased exercise intolerance, a fever, and increased white blood cells C) An 81-year-old male who has a productive cough and recurrent respiratory infections D) An 88-year-old female who experiences acute shortness of breath and airway constriction when exposed to tobacco smoke

C

Which of the following statements made by parents of high schoolers would be a cause for the concern the child may be thinking about suicide? A) "My child seems to eat all the time. He tells me that all of his friends are eating a lot as well." B) "My child seems to go shopping at the mall every day after school with her friends. I think they hang out at the mall." C) "My child has never had problems in school until now. He is failing classes and getting in trouble." D) "My child used to talk to me about anything. Now she spends most of her time in her room texting friends."

C

Which of the following statements provides blood work results and rationale that would be most closely associated with acute coronary syndrome? A) Increased serum creatinine and troponin I as a result of enzyme release from damaged cells B) Increased serum potassium and decreased sodium as a result of myocardial cell lysis, release of normally intracellular potassium, and disruption of the sodium-potassium pump C) Elevated creatine kinase and troponin, both of which normally exist intracellularly rather than in circulation D) Low circulatory levels of myoglobin and creatine kinase as a result of the inflammatory response

C

Which of the following teaching points would be most appropriate with a client who has a recent diagnosis of von Willebrand disease? A) "It's important that you avoid trauma." B) "Your disease affects your platelet function rather than clot formation." C) "Make sure that you avoid taking aspirin." D) "Clotting factor VIII can help your body compensate for the difficulty in clotting."

C

Your ESRD patient is receiving 2 units of packed red blood cells for anemia (Hgb of 8.2). Twenty minutes into the first transfusion, the nurse observes the patient has a flushed face, hives over upper body trunk, and is complaining of pain in lower back. His vital signs include pulse rate of 110 and BP drop to 95/56. What is the nurse's priority action? A) Slow the rate of the blood infusion to 50 mL/hour. B) Document the assessment as the only action. C) Discontinue the transfusion and begin an infusion of normal saline. D) Recheck the type of blood infusing with the chart documentation of patient's blood type.

C

Which of the following changes associated with aging contributes to heart failure development in older adults? Select all that apply. A) Increased incidence of mitral stenosis B) Sludge buildup in the kidneys C) Elevated diastolic BP D) Increased vascular stiffness E) Inflammation in the joints due to arthritis

C D

A physician is providing care for a child who has a diagnosis of cystic fibrosis (CF). Place the following events in the etiology of CF in ascending chronological order. Use all the options. A) Airway obstruction B) Recurrent pulmonary infections C) Impaired Cl- transport D) Decreased water content of mucociliary blanket E) Increased Na+ absorption

C E D A B

A 22-year-old female who adheres to a vegan diet has been diagnosed with iron deficiency anemia. Which of the following components of her diagnostic blood work would be most likely to necessitate further investigation? A) Decreased mean corpuscular volume (MCV) B) Decreased hemoglobin and hematocrit C) Microcytic, hypochromic red cells D) Decreased erythropoietin levels

D

A 31-year-old woman with a congenital heart defect reports episodes of light-headedness and syncope, with occasional palpitations. A resting electrocardiogram reveals sinus bradycardia, and she is suspected to have sick sinus syndrome. Which of the following diagnostic methods is the best choice to investigate the suspicion? A) Signal-averaged ECG B) Exercise stress testing C) Electrophysiologic study D) Holter monitoring

D

A 36-year-old woman with a diagnosis of antiphospholipid syndrome is receiving a scheduled checkup from her nurse practitioner. Which of the following teaching points would the nurse most likely prioritize? A) "It's important for you to do regular physical activity and maintain a healthy body weight." B) "Good nutrition and blood sugar control are important in your case." C) "You'll need to avoid taking nonsteroidal anti-inflammatory drugs when you have menstrual cramps." D) "You need to ensure your birth control pills don't contain estrogen."

D

A 53-year-old man presents with inability to concentrate, itching in his fingers and toes, elevated blood pressure, and unexplained weight loss. He is diagnosed with primary polycythemia. What will be the primary goal of his treatment? A) To control his hypertension B) To increase the amount of oxygen distributed by his red blood cells C) To reduce the mean size of his red cells D) To reduce the viscosity of his blood

D

A 54-year-old man with a long-standing diagnosis of essential hypertension is meeting with his physician. The patient's physician would anticipate that which of the following phenomena is most likely occurring? A) The patient's juxtaglomerular cells are releasing aldosterone as a result of sympathetic stimulation. B) Epinephrine from his adrenal gland is initiating the renin-angiotensin-aldosterone system. C) Vasopressin is exerting an effect on his chemoreceptors and baroreceptors resulting in vasoconstriction. D) The conversion of angiotensin I to angiotensin II in his lungs causes increases in blood pressure and sodium reabsorption.

D

A 63-year-old male client has been diagnosed with a bundle branch block. How will this client's care team most likely expect his condition to be expressed diagnostically? A) His AV node will be performing the primary pacemaker role due to inadequacy of the SA node. B) His ECG will show a flattened P wave as a result of impaired atrial depolarization. C) Conduction from the Purkinje fibers to the bundle branches is compromised by inadequate conduction. D) His ECG will show an inordinately wide QRS complex because impulses are bypassing the normal conduction tissue.

D

A 66-year-old obese man with diagnoses of ischemic heart disease has been diagnosed with heart failure that his care team has characterized as attributable to systolic dysfunction. Which of the following assessment findings is inconsistent with his diagnosis? A) His resting blood pressure is normally in the range of 150/90, and an echocardiogram indicates his ejection fraction is 30%. B) His end-diastolic volume is higher than normal, and his resting heart rate is regular and 82 beats/minute. C) He is presently volume overloaded following several days of intravenous fluid replacement. D) Ventricular dilation and wall tension are significantly lower than normal.

D

A 70-year-old woman with ongoing severe atrial fibrillation is scheduled for defibrillation. What is an aspect of the rationale and physiology of defibrillation treatment? A) Interruption of disorganized impulses by the current allows the AV node to readopt its normal pacemaker role. B) Defibrillation can be achieved using either a transcutaneous or transvenous pacemaker. C) Defibrillation must be coincided with the R wave of the ECG in order to be successful. D) The goal is to depolarize the entire heart during the passage of current.

D

A 77-year-old lifetime smoker has been diagnosed with a tumor in his lung at the site of an old tubercle scarring site, located in a peripheral area of his bronchiolar tissue. What is this client's most likely diagnosis? A) Squamous cell carcinoma B) Small cell lung cancer C) Large cell carcinoma D) Adenocarcinoma

D

A client with a diagnosis of hemolytic anemia has gone to a community-based laboratory for follow-up blood work. The lab technician confirms with the client that hematocrit is one of the components of the blood work. The client replies, "I thought the point of the blood work was to see how many red blood cells I have today." How could the technician best respond to the client's statement? A) "This result will tell your care provider about the number of red blood cells in a given quantity of your blood plasma." B) "Your hematocrit measures the average size of your red blood cells and indirectly measures your oxygen-carrying capacity." C) "The result will indicate how many of your red blood cells are new and young and will indicate your body's production rate of red cells." D) "The hematocrit measures the mass that your red blood cells account for in a quantity of your blood."

D

A client with a gastrointestinal bleed secondary to alcohol abuse and a hemoglobin level of 5.8 g/dL has been ordered a transfusion of packed red blood cells. The client possesses type B antibodies but lacks type D antigens on his red cells. Transfusion of which of the following blood types would be least likely to produce a transfusion reaction? A) B- B) B+ C) A+ D) A-

D

A community health nurse is conducting a class on the nutritional component for new mothers. Which of the following teaching points would be most justified? A) "Iron supplementation is not necessary provided you are breast-feeding your infant." B) "Be aware that cow's milk depletes your baby's supply of iron." C) "Your infant needs the same amount of iron as you but has far fewer sources for obtaining it." D) "If you choose to feed your baby with formula, ensure that it is iron fortified."

D

A medical student is familiarizing herself with recent overnight admissions to an acute medical unit of a university hospital. Which of the following patients would the student recognize as least likely to have a diagnosis of antiphospholipid syndrome in his or her medical history? A) A 66-year-old obese male with left-sided hemiplegia secondary to a cerebrovascular accident B) A 90-year-old female resident of a long-term care facility who has been experiencing transient ischemic attacks C) A 30-year-old female with a diagnosis of left leg deep vein thrombosis and a pulmonary embolism D) A 21-year-old male with a diagnosis of cellulitis and suspected endocarditis secondary to intravenous drug use

D

A new mother and father are upset that their 2-day-old infant is requiring phototherapy for hyperbilirubinemia. The pediatrician who has followed the infant since birth is explaining the multiplicity of factors that can contribute to high serum bilirubin levels in neonates. Which of the following factors would the physician be most likely to rule out as a contributor? A) The fact that the infant is being breast-fed B) Hypoxia C) Hepatic immaturity of the infant D) Transitioning of hemoglobin F (HbF) to hemoglobin A (HbA)

D

A number of clients have presented to the emergency department in the last 32 hours with complaints that are preliminarily indicative of myocardial infarction. Which of the following clients is least likely to have an ST-segment myocardial infarction (STEMI)? A) A 70-year-old woman who is complaining of shortness of breath and vague chest discomfort B) A 66-year-old man who has presented with fatigue, nausea and vomiting, and cool, moist skin C) A 43-year-old man who woke up with substernal pain that is radiating to his neck and jaw D) A 71-year-old man who has moist skin, fever, and chest pain that is excruciating when he moves but relieved when at rest

D

A nurse is providing care for a client who has a history of severe atherosclerosis. Which of the following clinical manifestations of the client's illness should the nurse anticipate and assess in the client? A) Motor deficits in muscles distal to plaque formation B) Peripheral vasodilation to compensate for ischemia C) Cognitive deficits due to ischemia or thrombosis D) Aneurysm formation due to weakening of blood vessel walls E) Necrosis of the vessel wall

D

A nurse practitioner is instructing a group of older adults about the risks associated with high cholesterol. Which of the following teaching points should the participants try to integrate into their lifestyle after the teaching session? A) "Remember the 'H' in HDL and the 'L' in LDL correspond to high danger and low danger to your health." B) "Having high cholesterol increases your risk of developing diabetes and irregular heart rate." C) "Smoking and being overweight increases your risk of primary hypercholesterolemia." D) "Your family history of hypercholesterolemia is important, but there are things you can do to compensate for a high inherited risk."

D

A nursing instructor is explaining the role of vascular smooth muscle cells in relation to increases in systemic circulation. During discussion, which neurotransmitter is primarily responsible for contraction of the entire muscle cell layer thus resulting in decreased vessel lumen radius? A) Nitric oxide B) Adrenal glands C) Fibroblast growth factor D) Norepinephrine

D

A patient has been diagnosed with anemia. The physician suspects an immune hemolytic anemia and orders a Coombs test. The patient asks the nurse what this test will tell the doctor. The nurse replies, A) "They will wash your RBCs and then mix the cells with a reagent to see if they clump together." B) "They will look at your RBCs under a microscope to see if they have an irregular shape (poikilocytosis)." C) "They will be looking to see if you have enough ferritin in your blood." D) "They are looking for the presence of antibody or complement on the surface of the RBC."

D

A physiotherapist is measuring the lying, sitting, and standing blood pressure of a patient who has been admitted to hospital following a syncopal episode and recent falls. Which of the following facts about the patient best relates to these health problems? A) The patient is male and has a history of hypertension. B) The patient's cardiac ejection fraction was 40% during his last echocardiogram. C) The patient has a history of acute and chronic renal failure. D) The client is 89 years old and takes a diuretic medication for his congestive heart failure.

D

A woman has recently determined that she is pregnant, and her clinician believes that the conception occurred around 8 weeks prior. Since the embryo is in the third stage of embryonic development, which of the following events and processes in growth and development would be expected to be taking place? A) Transition from a morula to a blastocyst B) Ossification of the skeleton and acceleration of body length growth C) Rapid eye movement and early support of respiration D) Formation of upper limbs and opening of the eyes

D

A young woman has been diagnosed by her family physician with primary Raynaud disease. The woman is distraught stating, "I've always been healthy, and I can't believe I have a disease now." What would be her physician's most appropriate response? A) "This likely won't have a huge effect on your quality of life, and I'll prescribe anticlotting drugs to prevent attacks." B) "I'll teach you some strategies to minimize its effect on your life, and minor surgery to open up your blood vessels will help too." C) "You need to make sure you never start smoking, and most of the symptoms can be alleviated by regular physical activity." D) "If you make sure to keep yourself warm, it will have a fairly minimal effect; I'll also give you pills to enhance your circulation."

D

Amniocentesis has suggested that a couple's first child will be born with sickle cell disease. The parents are unfamiliar with the health problem, and their caregiver is explaining the complexities. Which of the following statements by the parents would suggest a need for further teaching or clarification? A) "Our baby's red cells are prone to early destruction because of his or her weak membranes." B) "Not all of his or her red cells will be sickled, but low oxygen levels can cause them to become so." C) "Sickled cells can block his or her blood vessels, especially in the abdomen, chest, and bones." D) "Our son or daughter likely won't show the effects of sickling until he or she is school-aged because of the different hemoglobin in babies."

D

An 81-year-old male resident of a long-term care facility has a long-standing diagnosis of heart failure. Which of the following short-term and longer-term compensatory mechanisms are least likely to decrease the symptoms of his heart failure? A) An increase in preload via the Frank-Starling mechanism B) Sympathetic stimulation and increased serum levels of epinephrine and norepinephrine C) Activation of the renin-angiotensin-aldosterone system and secretion of brain natriuretic peptide (BNP) D) AV node pacemaking activity and vagal nerve suppression

D

An autopsy is being performed on a 44-year-old female who died unexpectedly of heart failure. Which of the following components of the pathologist's report is most suggestive of a possible history of poorly controlled blood pressure? A) "Scarring of the urethra suggestive of recurrent urinary tract infections is evident." B) "Bilateral renal hypertrophy is noted." C) "Vessel wall changes suggestive of venous stasis are evident." D) "Arterial sclerosis of subcortical brain regions is noted."

D

Around 3 weeks after razing an old chicken house, a 71-year-old retired farmer has developed a fever, nausea, and vomiting. After ruling out more common health problems, his care provider eventually made a diagnosis of histoplasmosis. Which of the following processes is most likely taking place? A) Toxin production by Histoplasma capsulatum is triggering an immune response. B) Antibody production against the offending fungi is delayed by the patient's age and the virulence of the organism. C) Spore inhalation initiates an autoimmune response that produces the associated symptoms. D) Macrophages are able to remove the offending fungi from the bloodstream but can't destroy them.

D

As part of a public health initiative, a nurse is teaching a group of older adults about ways to promote and maintain their health. Recognizing that the common cold is a frequent source of ailment, the nurse is addressing this health problem. Which of the following teaching points about the common cold is most accurate? A) "You shouldn't be taking antibiotics for a cold until your doctor has confirmed exactly which bug is causing your cold." B) "It's important to both cover your mouth when you cough or sneeze and encourage others to do so, since most colds are spread by inhaling the germs." C) "Scientists don't yet know exactly what virus causes the cold, and there is not likely to be a vaccine until this is known." D) "Use caution when choosing over-the-counter drugs for your cold; most people do best with rest and antifever medications."

D

In which of the following patients, would diagnostic investigations least likely reveal increased thrombopoietin production? A) An 81-year-old woman with diagnoses of rheumatoid arthritis and failure to thrive B) A 55-year-old man with dehydration secondary to Crohn disease C) A 66-year-old woman with a diagnosis of lung cancer with bone metastases D) A 21-year-old woman awaiting bone marrow transplant for myelogenous leukemia

D

Mr. V. has been admitted for exacerbation of his chronic heart failure (HF). When the nurse walks into his room, he is sitting on the edge of the bed, gasping for air, and his lips are dusty blue. Vital signs reveal heart rate of 112, respiratory rate of 36, and pulse oximeter reading of 81%. He starts coughing up frothy pink sputum. The priority intervention is to A) have medical supply department bring up suction equipment. B) apply oxygen via nasal cannula at 3 lpm. C) page the respiratory therapist to come give him a breathing treatment. D) call for emergency assistance utilizing hospital protocol.

D

The exasperated parents of a 4-month-old infant with colic have asked their health care provider what they can do to alleviate their child's persistent crying. Based on their concerns, the nurse should educate/discuss with the parents which of the following? A) Encouraging them to walk away from the infant when they can no longer tolerate it B) Recommending them to reduce the amount of commercial formula and increase breast-feeding C) Discussing the use of prescribed antiflatulent medication that will help more than changing the formula D) Demonstrating how to use a soothing voice and slow rocking back and forth as a way to calm the infant

D

The school nurse knows several children with hemophilia A. After recess, one student with hemophilia comes to the school nurse complaining of pain in his knee from falling on the playground. The nurse notes there is swelling in the knee and pain on palpation. The nurse should A) administer some NSAIDs to relieve the pain. B) wrap the knee in an ace bandage for compression. C) apply some warm compresses to the knee. D) notify parents to pick up the child and possibly administer factor VIII.

D

When a 55-year-old patient's routine blood work returns, the nurse notes that his C-reactive protein (CRP) is elevated. The patient asks what that means. The nurse responds, A) "You must eat a lot of red meat since this means you have a lot of fat floating in your vessels." B) "You are consuming high levels of folate, which works with the B vitamins and riboflavin to metabolize animal protein." C) "This means you have high levels of HDL to balance the LDL found in animal proteins." D) "This means you have elevated serum markers for systemic inflammation that has been associated with vascular disease

D

Which of the following infants most likely requires medical intervention? A) A 2-day-old baby boy who has caput succedaneum B) An infant 4 hours postpartum who has visible coning of his head following vaginal delivery C) A girl 3 days postpartum with noticeable unilateral cephalhematoma D) A male infant whose vertex delivery resulted in a brachial plexus injury

D

Which of the following situations related to transition from fetal to perinatal circulation would be most likely to necessitate medical intervention? A) Pressure in pulmonary circulation and the right side of the infant's heart fall markedly. B) Alveolar oxygen tension increases causing reversal of pulmonary vasoconstriction of the fetal arteries. C) Systemic vascular resistance and left ventricular pressure are both increasing. D) Pulmonary vascular resistance, related to muscle regression in the pulmonary arteries, rises over the course of the infant's first week.

D

Which of the following situations would be most deserving of a pediatrician's attention? A) The mother of an infant 2 days postpartum notes that her baby has intermittent periods of hyperventilation followed by slow respirations or even brief periods of apnea. B) A volunteer in the nursery notes that one of the infants, aged 2 weeks, appears unable to breathe through his mouth, even when his nose is congested. C) A neonate is visibly flaring her nostrils on inspiration. D) A midwife notes that a newborn infant's chest is retracting on inspiration and that the child is grunting.

D

Which of the following statements best captures the etiology of the acute response phase of extrinsic (atopic) asthma? A) IgG production is heightened as a consequence of exposure to an allergen. B) Airway remodeling results in airflow limitations. C) Epithelial injury and edema occur along with changes in mucociliary function. D) Chemical mediators are released from presensitized mast cells.

D

While rock climbing, a 22-year-old male has endured a severe head injury. Which of the following statements best captures expected clinical manifestations and treatments for his immediate condition? A) Oxygen therapy is likely to decrease his respiratory drive and produce an increase in PCO2. B) Cheyne-Stokes breathing is likely but will respond to bronchodilators. C) The client is unlikely to respond to supplementary oxygen therapy due to impaired diffusion. D) Hypoventilation may exist, resulting in increased PCO2 and hypoxemia that may require mechanical ventilation

D

Following electrophysiological testing that included ablation therapy, the nurse should be assessing the patient for which complication that may occur postprocedure? A) Complaints of nausea and spitting up bile-looking secretions along with stomach cramps B) Sudden onset of dysonea, tachypnea, and chest pain of a "pleuritic" nature (worsened by breathing) C) Bleeding from the nose that requires packing, excessive swallowing of mucus, and coughing D) Complaints of heart palpitations, frequent PVCs noted on monitor, and substernal chest pain

V


Conjuntos de estudio relacionados

Accounting Fundamentals Sessions 7-13

View Set

Linux Chapter 1, Linux Test chapters 4-9

View Set

Intro to Computer Science Quizes

View Set

MKT 300: Chapter 16: Retailing and Omnichannel Marketing

View Set

Calculation of maintenance fluids

View Set

Chapter 27: Lower Respiratory Problems

View Set

Pharm Ch 3: Toxic Effects of Drugs

View Set

Chapter 35: Assessment of Immune Function

View Set

testout cyberdefense pro chapter 10

View Set